Megan leaves her house at 4:15 to go soccer practice. It takes her 35 minutes to get there. Her practice is two hours long. Then, she drives home, which takes 40 minutes. What time does she get back home?

Answers

Answer 1

Answer:

7:35

Step-by-step explanation:

we take the 35 and 45 and add it together, then take out the 60 minutes and put that in as an hour. the practice is two hours long plus the hour we took out. then the remaining minutes are 20. we add 20 minutes and three hours


Related Questions

how many 6-digit numbers can be created using8, 0, 1, 3, 7, and 5 if each number is used only once?

Answers

Answer:

600 numbers

Step-by-step explanation:

For six-digit numbers, we need to use all digits 8,0,1,3,7,5 each once.

However, 0 cannot be used as the first digit, because it would make a 5-digit number.

Therefore

there are 5 choices for the first digit (exclude 0)

there are 5 choices for the first digit (include 0)

there are 4 choices for the first digit

there are 3 choices for the first digit

there are 2 choices for the first digit

there are 1 choices for the first digit

for a total of 5*5*4*3*2*1 = 600 numbers

If a polygon has an area of 10 cm² and is dilated by a factor of 2, what will be the area of the dilated polygon?

Answers

Area depends on the product of sides,

so if the sides are shortened by a factor of 2, area will reduce by a factor of 4. (2×2)

new area = 10/4=2.5 cm²

7 liters of gasoline between their 4 cars. How many liters of gasoline should each car get?

Answers

Answer:

1.75 liters for each car.

Step-by-step explanation:

Divide:

*7 / 4 = 1.75.

Check:

*1.75 x 4 = 7.

1.75
7 divided by 4

Which equation represents the line that is perpendicular to y=3/4x+1 and passes through (-5,11)
Will give brainliest!!

Answers

Answer:

y = - [tex]\frac{4}{3}[/tex] x + [tex]\frac{13}{3}[/tex]

Step-by-step explanation:

The equation of a line in slope- intercept form is

y = mx + c ( m is the slope and c the y- intercept )

y = [tex]\frac{3}{4}[/tex] x + 1 ← is in slope- intercept form

with slope m = [tex]\frac{3}{4}[/tex]

Given a line with slope m then the slope of a line perpendicular to it is

[tex]m_{perpendicular}[/tex] = - [tex]\frac{1}{m}[/tex] = - [tex]\frac{1}{\frac{3}{4} }[/tex] = - [tex]\frac{4}{3}[/tex] , thus

y = - [tex]\frac{4}{3}[/tex] x + c ← is the partial equation

To find c substitute (- 5, 11) into the partial equation

11 = [tex]\frac{20}{3}[/tex] + c ⇒ c = 11 - [tex]\frac{20}{3}[/tex] = [tex]\frac{13}{3}[/tex]

y = - [tex]\frac{4}{3}[/tex] x + [tex]\frac{13}{3}[/tex] ← equation of perpendicular line

The equation of the line that passes through (-5, 11) and perpendicular to y = (3/4)x + 1 is

y = -2x + 1

What is an equation of a line?

The equation of a line is given by:

y = mx + c

where m is the slope of the line and c is the y-intercept.

Example:

The slope of the line y = 2x + 3 is 2.

The slope of a line that passes through (1, 2) and (2, 3) is 1.

We have,

y = (2/4)x + 1 is in the form of y = m(2)x + c

So,

m(2) = 2/4 = 1/2

The equation of the line y = m(1)x + c is perpendicular to y = (2/4)x + 1.

So,

m(1) x m(2) = -1

m(1) = -1/(1/2)

m(1) = -2

Now,

y = -2x + c passes through (-5, 11).

This means,

11 = -2 x (-5) + c

11 = 10 + c

11- 10 = c

c = 1

Thus,

The equation of the line is y = -2x + 1.

Learn more about equation of a line here:

https://brainly.com/question/23087740

#SPJ2

The numbers in the select boxes are 4 7 and 3 I put them with the photos. Can someone help

Answers

Answer:

4:7

Step-by-step explanation:

4 grape candies : 7 total candies (grape + cherry)

Answer:

It is simply 4:3

Anyone want to help...?

Answers

Answer:

-1

Step-by-step explanation:

3/2 * (-22/33)

Simplify by dividing the second fraction by 11

3/2 * (-2/3)

Rewriting

3/3 * (-2/2)

-1/1

Answer:

-1

Step-by-step explanation:

(a/b)(c/d) = (a*c)(

(3/2)(-22/33)

(3*-22)/(2*33) = -66/66 = -1

Solve the simultaneous equation
X+3y=13
X-y=5

Answers

Answer:

x = 7

y = 2

Step-by-step explanation:

In the above question, we are given 2 equations which are simultaneous. To solve this equation, we have to find the values of x and y

x + 3y = 13 ........ Equation 1

x - y = 5...........Equation 2

From Equation 2,

x = 5 + y

Substitute 5 + y for x in Equation 1

x + 3y = 13 ........ Equation 1

5 + y + 3y = 13

5 + 4y = 13

4y = 13 - 5

4y = 8

y = 8/4

y = 2

Since y = 2, substitute , 2 for y in Equation 2

x - y = 5...........Equation 2

x - 2 = 5

x = 5 + 2

x = 7

Therefore, x = 7 and y = 2

find the lower quartile for the data {47.2, 33.8, 43, 62, 5.8, 9, 61.4, 30.8, 68.2, 51.6, 13.2, 17.4, 64.2, 50.6, 29.4, 40.4}

Answers

Answer:

The lower quartile is 23.4

Step-by-step explanation:

The given data are;

47.2, 33.8, 43, 62, 5.8, 9, 61.4, 30.8, 68.2, 51.6, 13.2, 17.4, 64.2, 50.6, 29.4, 40.4

Rearranging the data, we have;

5.8, 9, 13.2, 17.4, 29.4, 30.8, 33.8, 40.4, 43, 47.2, 50.6, 51.6, 61.4, 62, 64.2, 68.2

The lower quartile, Q₁, is the (n + 1)/4 th term which is (16 +1)/4 = 4.25th term

However since we have an even set of numbers, we place a separator at the middle and we look for the median of the left half as follows

5.8, 9, 13.2, 17.4, 29.4, 30.8, 33.8, 40.4║ 43, 47.2, 50.6, 51.6, 61.4, 62, 64.2, 68.2

We have two numbers (17.4 + 29.4) at the median of the left set of numbers, we find the average of the two numbers to get the lower quartile

The lower quartile is therefore = (17.4 + 29.4)/2 = 23.4.

What is the width of the rectangle shown below?
4x + 3
A = 8x2 – 10x – 12

Answers

Answer:

2x-4

Step-by-step explanation:

Area of a rectangle = Length * Width

Given parameters

Area A = 8x2 – 10x – 12

Length of the rectangle = 4x+3

Required

Width of the rectangle.

Substituting the given parameters into the formula

8x2 – 10x – 12  = (4x+3)*width

width = 8x2 – 10x – 12 /4x+3

S

Factorizing the numerator

8x² – 10x – 12

= 2(4x²-5x-6)

= 2(4x²-8x+3x-6)

= 2(4x(x-2)+3(x-2))

= 2(4x+3)(x-2)

Width = 2(4x+3)(x-2)/4x+3

Width = 2(x-2)

Width = 2x-4

Hence the width of the rectangle is 2x-4

Which of the following systems has (1,−1) as a solution? 1. 3x−2y=1 5x+3y=8 2. 3x+2y=1 5x+3y=−8 3. 3x+2y=1 5x−3y=8 4. 2x+3y=1 5x−3y=−8

Answers

2. 3x+2y= 1 and 5x+3y=-8

solution: In picture.

can u help me with this?​

Answers

Answer:  Yes. The sales tax is 5% which equals $4.20 for $84

Step-by-step explanation:

[tex]\dfrac{0.60}{12}=0.05\qquad \rightarrow 5\%\\\\\\\dfrac{1.20}{24}=0.05\qquad \rightarrow 5\%\\\\\\\dfrac{1.80}{36}=0.05\qquad \rightarrow 5\%\\\\\\\dfrac{2.40}{48}=0.05\qquad \rightarrow 5\%[/tex]

The sales tax rate is proportional for the values in the table.

$84 x 0.05 = $4.20

The sales tax on a purchase of $84 is $4.20

Dawn and Jackson have baseball cards in the ratio of 2:3. Together, they have a total of 60 baseball cards. How many baseball cards does each child have?

Answers

Answer:

24 and 36

Step-by-step explanation:

2x + 3x = 60

5x = 60

x = 12

Dawn has 2(12) = 24

Jackson has 3(12) = 36

Step-by-step explanation:

To find the number of baseball cards each person received we must first find the total parts

That's

2 + 3 = 5

For Dawn

Dawn's part is 2

We have

2/5 × 60

= 24 baseball cards

For Jackson

Jackson's part is 3

That's

3/5 × 60

= 36 baseball cards

Hope this helps you

Suppose y varies jointly as x & z. If y = -180 when z = 15and x = -3,then find y when x = 7 and z = -5. 1 point

Answers

Answer:

y = - 140

Step-by-step explanation:

The statement

y varies jointly as x & z is written as

y = kxz

to find y when x = 7 and z = -5 we must first find the relationship between the variables

when y = - 180

z = 15

x = - 3

- 180 = k(15)(-3)

-180 = - 45k

Divide both sides by - 45

k = 4

The formula for the variation is

y = 4xz

when

x = 7

z = -5

y = 4(7)(-5)

y = 4(-35)

y = - 140

Hope this helps you

[fill in the blank]
In this figure,AB and CD are parallel.

AB is perpendicular to line segment_____. If the length of EF is a units, then the length of GH is_____units.

Answers

Answer:

1. GH

2. a

Step-by-step explanation:

Perpendicular: When 2 lines meet at 90 degrees

1. It is line segment GH because AB and GH meet at a 90 degree angle (since there is a box at angle GHF indicating that it is 90 degrees)

2. It has to be a units because it is a rectangle where the top and bottom are congruent and the sides are too

This is a rectangle since AB and CD are parallel and GH can be a transversal line, according to same side interior angles theorem EGH is a also 90 degrees. That means FEG is 90 degrees too because then the quadrilateral will add up to 360 degrees

Puzzle corner
Look Before You Leap!
See how long it takes you to work out the
following:
(1 x2)×(3 x 4)×(586)×(7 x 8) x (
9×0)

Answers

Answer:

0

Step-by-step explanation:

Notice that the last factor is null (9×0)

So the result will be null since any number that is multiplied by 0 equals 0.

Find the slope of the line that passes through the points (-8,-3) and (2, 3)
0
1
3/5
5/3

Answers

Answer:

The answer is

[tex] \frac{3}{5} [/tex]

Step-by-step explanation:

To find the slope passing through two points we use the formula

[tex]m = \frac{y2 - y1}{x2 - x1} [/tex]

Where

m is the slope

( x1 , y1) and ( x2 , y2) are the points

From the question the points are

(-8,-3) and (2, 3)

So the slope is

[tex]m = \frac{3 + 3}{2 + 8} = \frac{6}{10} = \frac{3}{5} [/tex]

Hope this helps you

Will Give Brainliest, Answer ASAP m∠O =
m∠N =

Answers

Answer:

∠ O = 61°, ∠ N = 119°

Step-by-step explanation:

In a parallelogram

Consecutive angles are supplementary

Opposite angles are congruent, thus

x + 2x - 3 = 180

3x - 3 = 180 ( add 3 to both sides )

3x = 183 ( divide both sides by 3 )

x = 61°

Thus

∠ O = ∠ M = x = 61°

∠ N = ∠ P = 2x - 3 = 2(61) - 3 = 122 - 3 = 119°

HELP i don’t know how to do this

Answers

Answer:

4a

Step-by-step explanation:

4a

the top and right are a-b, but you have to add the b’s back in, so really all sides are a

a+a+a+a=4a

Answer: 4a

Step-by-step explanation: perimeter is the length of all sides added together. Every length is given combine like variables and you will get 4a+2b-2b. 2b-2b is 0 which leaves you with 4a

Find the square root of 2601 by prime factorization (USE MULTIPLICATION METHOD TO SOLVE THE ABOVE QUESTION)

Answers

2601|3

867|3

289|17

17|17

1

[tex]\sqrt{2601}=\sqrt{3^2\cdot17^2}=3\cdot17=51[/tex]

You own a farm and have several fields in which your livestock grazes. You need to order barbed-wire fencing for a small pasture that has a length of 5 yards and a width of 3 yards. The barbed wire must be long enough to be placed on all four sides of the outside pasture. How many yards of barbed-wire should you order?

Answers

Answer:

16 yards of barbed wire

Step-by-step explanation:

Length=5 yards

Width=3 yards

Perimeter of the pasture=2(length + width)

=2(5 yards +3 yards)

=2(8 yards)

=16 yards

You should order 16 yards of barbed wire for fencing the pasture

Write an equation for the line in the graph that passes through the points (0,4) and (12,16).

Answers

Answer:

We have,

y-y1=m(x-x1)

or,y-4=-1(x-0)

or,y-4=-x

or,x+y=4 is the required equation

Step-by-step explanation:

it it helps u ...plz mark it as brainliest

In the first quadrant you start at 5, 6 and move 4 units down. What point will you end up at? Thanks for your help! - Someone who's better at English than math

Answers

Answer:

(5, 2)

Step-by-step explanation:

(5, 6) go down 4 units means subtract 4 from the y

(5, 2)

The point to end up will be (5, 2).

What is Coordinates?

A pair of numbers which describe the exact position of a point on a cartesian plane by using the horizontal and vertical lines is called the coordinates.

Given that;

In the first quadrant you start at (5, 6 ) and move 4 units down.

Now,

Since, In the first quadrant you start at 5, 6 and move 4 units down.

Hence, The end up point = (5, 6 - 4)

                                       = (5, 2)

Thus, The point to end up will be (5, 2).

Learn more about the coordinate visit:

https://brainly.com/question/18269861

#SPJ5

PLS HELP. i really need this fast ill give brainliest too

Answers

Answer:

24 square units

Step-by-step explanation:

Use the formula for area of a parallelogram to solve.  The base is 6 units, and the height is 4 units.

A = bh

A = (6)(4)

A = 24 square units

The area of the parallelogram is 24 square units.

(b) The train is 61 cm long and travels at a speed of 18 cm/s.
It takes 4 seconds for the whole of the train to cross a bridge.
Calculate the length of the bridge.​

Answers

Answer:

The length of the bridge is 72 cm

Step-by-step explanation:

In order to find the length of bridge, we have to apply distance formula which is D = S × T where S represents speed and T is time :

[tex]d = s \times t[/tex]

[tex]let \: s = 18,t = 4[/tex]

[tex]d = 18 \times 4[/tex]

[tex]d = 72 \: cm[/tex]

The length of the bridge is 11 cm .

What is relationship between distance time and speed ?

When an object moves in a straight line at a steady speed, we can calculate its speed if we know how far it travels and how long it takes. This equation shows the relationship between speed, distance traveled and time taken:

Speed is distance divided by the time taken.

For example, a car travels 30 kilometers in 2 hours.

Its speed is 30 ÷ 2 = 15km/hr.

Formula used :

Distance  = Speed * Time

Time = Distance / Speed

Speed = Distance / Time

According to the question

Length of train = 61 cm

Speed of train = 18 cm/s

Time taken to cross the bridge = 4 seconds

In this length traveled by train = length of train + Length of bridge

( as time given is to completely cross platform )

Therefore,

length traveled by train = 61 + Length of bridge

formula used

Distance  = Speed * Time

61 + Length of bridge =  18 * 4

61 + Length of bridge = 72

Length of bridge  = 72 - 61

Length of bridge  = 11 cm

Hence, the length of the bridge is 11 cm .

To know more about relationship between distance time and speed  here :

https://brainly.com/question/26754391

# SPJ2

Please answer this question now

Answers

Answer:

AB = 72°

Step-by-step explanation:

The inscribed angle ADC is half the measure of its intercepted arc, thus

56° = [tex]\frac{1}{2}[/tex] ( m ABC ) ← multiply both sides by 2

112° = ABC

ABC = AB + BC = AB + 40, so

AB + 40 = 112 ( subtract 40 from both sides )

AB = 72°

The base of a triangle is two times its height. If the area of the triangle is 36, then what is the height of the triangle?

Answers

We have:

h - height

b = 2h - base

A = 36 - area

so:

[tex]A=\frac{1}{2}\cdot b\cdot h\\\\A=\frac{1}{2}\cdot 2\cdot h \cdot h\\\\A=h^2\\\\36=h^2\quad|\sqrt{(\dots)}\\\\\boxed{h=6}[/tex]

PLEASE HELP ME WORTH 20 POINTS It looks like the graph of the parents function f(x)x^2. However:
- It has been reflected (flipped) over the x-axis
-It has been shifted down 4 units.
-It had been shifted left 1 unit

Step 1: Start with the equation f(x) = x2. Write the equation for the graph of g(x) that has been reflected, or flipped, over the x-axis.

Step 2: Use the equation you wrote in Step 1. Write the equation for the graph of g(x) that has also been shifted down 4 units.

Step 3: Use the equation you wrote in Step 2. Write the equation for the graph of g(x) that has also been shifted left 1 unit.

Answers

flipped : [tex]-x^2[/tex]

moving down: [tex] -x^2+4[/tex]

shifting left [tex] -(x+1)^2+4[/tex]

expanding it: [tex] -x^2-2x+3[/tex]

Answer:

1. f(x)=x^2

f(x)=-x^2

2. f(x)=-x^2-4

3. f(x)=-(x+1)^2-4

Brian invested his savings in two investment funds. The $8000 that he invested in Fund A returned a 4% profit. The amount that he invested in Fund B returned a 1% profit. How much did he invest in Fund B, if both funds together returned a 2% profit?

Answers

Answer: Brian invested $16000 in Fund B .

Step-by-step explanation:

Let x be the amount Brian invested in Fund B.

Given, The $8000 that he invested in Fund A returned a 4% profit. The amount that he invested in Fund B returned a 1% profit.

i.e. profit on Fund A = 4% of 8000 = 0.04 ×8000 = $320

Profit on Fund B = 1% of x = 0.01x

Together they earn 1% profit, i.e. Combined profit = 2% of (8000+x)

= 0.02(8000+x)

As per question,

Combined profit=Profit on Fund A+Profit on Fund B

[tex]\Rightarrow\ 0.02(8000+x) =320+0.01x\\\\\Rightarrow\ 0.02(8000) +0.02x=320+0.01x\\\\\Rightarrow\ 160+0.02x=320+0.01x\\\\\Rightarrow\ 0.02x-0.01x=320-160\\\\\Rightarrow\ 0.01x=160\\\\\Rightarrow\ x=\dfrac{160}{0.01}\\\\\Rightarrow\ x=16000[/tex]

Hence, Brian invested $16000 in Fund B .

On a coordinate plane, a triangle has points (negative 5, 1), (2, 1), (2, negative 1).
Use the drop downs to answer the following questions about the distance between the points (−5, 1) and (2, −1).

What is the distance of the horizontal leg?

What is the distance of the vertical leg?

Use the Pythagorean theorem. What is the distance between the two points?

Answers

Answer:

The answer is below

Step-by-step explanation:

The points of the triangle are  (- 5, 1), (2, 1), (2, - 1). The distance between two points is given by:

[tex]Distance=\sqrt{(x_2-x_1)^2+(y_2-y_1)^2}[/tex]

The horizontal leg is formed by points with the same y axis. Therefore the points that make up the horizontal leg is (- 5, 1), (2, 1). The Distance of the horizontal leg is:

[tex]Horizontal\ leg=\sqrt{(2-(-5))^2+(1-1)^2}=\sqrt{7^2+0}=7\ units[/tex]

The vertical leg is formed by points with the same x axis. Therefore the points that make up the vertical leg is (2 1), (2, 1-). The Distance of the vertical leg is:

[tex]Vertical\ leg=\sqrt{(2-2)^2+(-1-1)^2}=\sqrt{0+(-2)^2}=2\ units[/tex]

The hypotenuse is gotten using Pythagorean theorem. It is gotten by:

Hypotenuse² = (Horizontal leg)² + (Vertical leg)²

Hypotenuse² = 7² + 2²

Hypotenuse² = 49 + 4 = 53

Hypotenuse = √53

Hypotenuse = 7.28 unit

Answer:

The answer are 7, 2 and 53

Step-by-step explanation:

convert the equation f(x)=1/2x^2+3x-2 to vertex form

Answers

Answer:

Step-by-step explanation:

Hello, please consider the following.

The "vertex form" is as below.

[tex]y=a(x-h)^2+k\\\\\text{Where (h, k) is the vertex of the parabola.}\\[/tex]

Let's do it!

[tex]f(x)=\dfrac{1}{2}x^2+3x-2\\\\f(x)=\dfrac{1}{2}\left(x^2+3*2*x\right) -2\\\\f(x)=\dfrac{1}{2}\left( (x+3)^2-3^2\right)-2\\\\f(x)=\dfrac{1}{2}(x+3)^2-\dfrac{9}{2}-\dfrac{4}{2}\\\\f(x)=\dfrac{1}{2}(x+3)^2-\dfrac{9+4}{2}\\\\\large \boxed{\sf \bf \ \ f(x)=\dfrac{1}{2}(x+3)^2-\dfrac{13}{2} \ \ }[/tex]

Thank you.

Other Questions
Please help! Suppose that [tex]\alpha[/tex] is inversely proportional to [tex]\beta[/tex]. If [tex]\alpha=4[/tex] when [tex]\beta=9[/tex], find [tex]\alpha[/tex] when [tex]\beta=-72[/tex] I need help answering these two questions Find an equation for the surface consisting of all points P in the three-dimensional space such that the distance from P to the point (0, 1, 0) is equal to the distance from P to the plane y Point B is on line segment AC. Given BC=9 and AB=11, determine the length AC. If 2x3 4x2 + kx + 10 is divided by (x + 2), the remainder is 4. Find the value of k using remainder theorem. Please help :) Bob has taken out a loan of $15,000 for a term of 48 months (4 years) at an interest rate of 6.5%. Using the amortization table provided, what will be his total financecharge over the course of his loan?Monthly Payment per $1,000 of PrincipalRate 1 Year 2 Years 3 Years 4 Years 5 Years6.5% $86.30 $44.55 $30.65 $23.71 $19.577.0% $86.53 $44.77$30.88$23.95$19.807.5% $86.76 $45.00$31.11$24.18$20.048.0% $86.99 $45.23$31.34$24.41$20.288.5% $87.22 $45.46$24.65$24.65$20.529.0% $87.45 $45.68$31.80$24.89$20.76A.$355.65OB.$975.00C.$1,682.40D.$2,071.20E. $17,071.20ResetNext 2020 Edmentum. All rights reserved. 6. How was France able to create an Enlightenment government at last under Napoleon's dictatorial rule?Napoleon included many ideals of the Enlightenment in the legal systems he created.The Treaty of Versailles included most of the ideals formed during the Enlightenment.The people of France copied the American Bill of Rights, and Napoleon applied its freedoms to all French citizens.Enlightenment ideals were written into the new French constitution, which was written by Napoleon. What happens when volcanoes release greenhouse gases into the atmosphere? Read the e-mail from Emily's mother, Teresa, about their planned trip. Create a response from Emily's perspective giving your opinions about howyou would like the trip to shape up.Querida EmilyHola, hija! Qu tal la Investigacin? Qu has aprendido? Yo siempre quise visitar las ruinas de civilizaciones antiguas como Machu Picchu oTeotihuacn. Pero quiero que t elijas el lugar para nuestras vacaciones. Me Interesa tambin el ecoturismo y me llaman la atencin los deportesde aventuras. Qu piensas?Espero que todo vaya bien, mi amor. Te extrao mucho. Dale recuerdos de mi parte a la familia.Con carino,Tu mam For the regression equation, = +20X + 200 what can be determined about the correlation between X and Y? In an addendum to the purchase and sale contract, the seller agrees to paint the house before conveying the house to the buyer. The weather changes, and the seller is unable to finish painting the house before closing. This is an example of _______. Hannah Co. has 10,000 shares of $10 par common stock outstanding. A 10% stock dividend is declared when the market price is $50 per share.Following the stock dividend, a cash dividend of $4 per share is declared and paid to Hannah Cos' shareholders. The debit to Retained Earnings will be: Jonas needs a cell phone. He has a choice between two companies with the following monthly billing policies. Each companys monthly billing policy has an initial operating fee and charge per text message. Sprint charges $29.95 monthly plus .15 cents per text, AT&T charges $4.95 monthly plus .39 cents per text. Create equations for the two cell phone plans. Data related to the inventories of Costco Medical Supply are presented below: Surgical Equipment Surgical Supplies Rehab Equipment Rehab Supplies Selling price $ 276 $ 134 $ 354 $ 152 Cost 156 136 255 152 Costs to sell 17 17 16 7 In applying the lower of cost or net realizable value rule, the inventory of surgical supplies would be valued at: A trader buys tea for $1200 and sells it for $1500. Per sack of tea he makes a profit of $50. How many sacks of tea did he have? In tests of a computer component, it is found that the mean time between failures is 520 hours. A modification is made which is supposed to increase the time between failures. Tests on a random sample of 10 modified components resulted in the following times (in hours) between failures. 518 548 561 523 536 499 538 557 528 563 At the 0.05 significance level, test the claim that for the modified components, the mean time between failures is greater than 520 hours. Use the P-value method of testing hypotheses.H0:H1:Test Statistic:Critical Value:Do you reject H0?Conclusion:If you were told that the p-value for the test statistic for this hypothesis test is 0.014, would you reach the same decision that you made for the Rejection of H0 and the conclusion as above? Let A represent going to the movies on Friday and let B represent going bowling on Friday night. The P(A) = 0.58 and the P(B) = 0.36. The P(A and B) = 94%. Lauren says that both events are independent because P(A) + P(B) = P(A and B) Shawn says that both events are not independent because P(A)P(B) P(A and B) Which statement is an accurate statement? Lauren is incorrect because the sum of the two events is not equal to the probability of both events occurring. Shawn is incorrect because the product of the two events is equal to the probability of both events occurring. Lauren is correct because two events are independent if the probability of both occurring is equal to the sum of the probabilities of the two events. Shawn is correct because two events are independent if the probability of both occurring is not equal to the product of the probabilities of the two events. Which equation will solve the following word problem? Jared has 13 cases of soda. He has 468 cans of soda. How many cans of soda are in each case? 13(468) = c 468c = 13 468/13 = c 13 = c/468 You go on a trip to the Galapagos island to study the animals there for a paper you are writing on evolution, just like Darwin did. You spend 3 days taking notes on all the physical characteristics you can observe about the tortoises and then you spend 3 days observing the finches. What have you recorded? A. Inherited characteristics B. Environmental variation factors C. Phenotypes D. Genotypes Disability and frustration can stop the progress of a Nation. How?